Last visit was: 24 Apr 2024, 23:45 It is currently 24 Apr 2024, 23:45

Close
GMAT Club Daily Prep
Thank you for using the timer - this advanced tool can estimate your performance and suggest more practice questions. We have subscribed you to Daily Prep Questions via email.

Customized
for You

we will pick new questions that match your level based on your Timer History

Track
Your Progress

every week, we’ll send you an estimated GMAT score based on your performance

Practice
Pays

we will pick new questions that match your level based on your Timer History
Not interested in getting valuable practice questions and articles delivered to your email? No problem, unsubscribe here.
Close
Request Expert Reply
Confirm Cancel
SORT BY:
Date
Tags:
Show Tags
Hide Tags
Most Helpful Reply
User avatar
Current Student
Joined: 18 Oct 2014
Posts: 680
Own Kudos [?]: 1763 [41]
Given Kudos: 69
Location: United States
GMAT 1: 660 Q49 V31
GPA: 3.98
Send PM
avatar
Intern
Intern
Joined: 26 May 2013
Posts: 42
Own Kudos [?]: 70 [6]
Given Kudos: 243
Send PM
General Discussion
Tutor
Joined: 16 Oct 2010
Posts: 14819
Own Kudos [?]: 64906 [3]
Given Kudos: 426
Location: Pune, India
Send PM
Re: Since smoking-related illnesses are a serious health problem [#permalink]
2
Kudos
1
Bookmarks
Expert Reply
WaitingSurprises wrote:
Since smoking-related illnesses are a serious health problem in Normark, and since addiction to nicotine prevents many people from quitting smoking, the government of Normark plans to reduce the maximum allowable quantity of nicotine per cigarette by half over the next five years. However, reducing the quantity of nicotine per cigarette will probably cause people addicted to nicotine to smoke more cigarettes. Therefore implementing this plan is unlikely to reduce the incidence of smoking related illnesses.

Which of the following, if true, most strongly supports the argument about the consequences of implementing the Normarkian government's plan.

(A) Over half of the nonsmoking adults in Normark have smoked cigarettes in the past.

(B) most cigarettes currently sold in Normark contain somewhat less than the maximum amount of nicotine permitted by law.

(C) Inexpensive, smoke-free sources of nicotine, such as nicotine gum and nicotine skin patches, have recently become available in Normark.

(D) Many smokers in Normark already spend a large proportion of their disposable income on cigarettes.

(E) The main cause of smoking-related illnesses is not nicotine but tar in cigarette smoke.


OA to follow after some discussion.


- Addiction to nicotine prevents many people from quitting
- Government plans to reduce the maximum allowable quantity of nicotine per cigarette.
- This will probably cause people addicted to nicotine to smoke more cigarettes.

Conclusion: Implementing this plan (reducing nicotine in cigs) is unlikely to reduce the incidence of smoking related illnesses.

What will strengthen the conclusion that the plan will not reduce illnesses?

(A) Over half of the nonsmoking adults in Normark have smoked cigarettes in the past.

How this number will be impacted because of the plan, we do not know.

(B) most cigarettes currently sold in Normark contain somewhat less than the maximum amount of nicotine permitted by law.

Irrelevant. The plan is to further reduce the amount of nicotine to reduce addiction.

(C) Inexpensive, smoke-free sources of nicotine, such as nicotine gum and nicotine skin patches, have recently become available in Normark.

How it will impact the sale of cigarettes, we don't know.

(D) Many smokers in Normark already spend a large proportion of their disposable income on cigarettes.

This implies that the people may not be able to buy more cigarettes. Hence, reduced nicotine consumption may reduce addiction. The plan may actually work. This is not what we are looking for!

(E) The main cause of smoking-related illnesses is not nicotine but tar in cigarette smoke.

Correct! People who continue smoking the same number of cigarettes with reduced nicotine are unlikely to get better health because tar causes health problems. Also, reducing nicotine may make people smoke more cigarettes leading to more tar consumption and hence more health problems. So the plan to reduce health problems may not actually work.


Answer (E)
Current Student
Joined: 24 Oct 2016
Posts: 166
Own Kudos [?]: 228 [0]
Given Kudos: 116
Location: India
Concentration: Technology, Strategy
GMAT 1: 710 Q49 V38
GMAT 2: 760 Q50 V44 (Online)
GPA: 3.61
Send PM
Re: Since smoking-related illnesses are a serious health problem [#permalink]
Hi expert,

Could you please let me know why Option C is incorrect? As per the argument, nicotine prevents many people from quitting smoking. And now as per this choice nicotine is going to be even more easily available. So, there is a good chance people get nicotine from another source - (despite the reduction of it in cigarettes.)

Many thanks in advance.
Target Test Prep Representative
Joined: 24 Nov 2014
Status:Chief Curriculum and Content Architect
Affiliations: Target Test Prep
Posts: 3480
Own Kudos [?]: 5137 [0]
Given Kudos: 1431
GMAT 1: 800 Q51 V51
Send PM
Re: Since smoking-related illnesses are a serious health problem [#permalink]
Expert Reply
jayarora wrote:
Hi expert,

Could you please let me know why Option C is incorrect? As per the argument, nicotine prevents many people from quitting smoking. And now as per this choice nicotine is going to be even more easily available. So, there is a good chance people get nicotine from another source - (despite the reduction of it in cigarettes.)

Many thanks in advance.

The point is that people addicted to nicotine smoke because, by doing so, they take in the nicotine present in the cigarettes they are smoking.

So, it’s not the availability of nicotine in general that causes people to smoke. It’s the presence of nicotine in cigarettes that does.

Posted from my mobile device
Intern
Intern
Joined: 11 Nov 2013
Posts: 45
Own Kudos [?]: 68 [0]
Given Kudos: 25
Send PM
Re: Since smoking-related illnesses are a serious health problem [#permalink]
Since smoking-related illnesses are a serious health problem in Normark, and since addiction to nicotine prevents many people from quitting smoking, the government of Normark plans to reduce the maximum allowable quantity of nicotine per cigarette by half over the next five years. However, reducing the quantity of nicotine per cigarette will probably cause people addicted to nicotine to smoke more cigarettes. Therefore implementing this plan is unlikely to reduce the incidence of smoking related illnesses.

Which of the following, if true, most strongly supports the argument about the consequences of implementing the Normarkian government's plan.

(A) Over half of the nonsmoking adults in Normark have smoked cigarettes in the past.

(B) most cigarettes currently sold in Normark contain somewhat less than the maximum amount of nicotine permitted by law.

(C) Inexpensive, smoke-free sources of nicotine, such as nicotine gum and nicotine skin patches, have recently become available in Normark.

(D) Many smokers in Normark already spend a large proportion of their disposable income on cigarettes.

(E) The main cause of smoking-related illnesses is not nicotine but tar in cigarette smoke.

VeritasKarishma GMATNinja

Are "nicotine skin patches" not a mode of smoking, thereby a strong case to consider option C?
GMAT Club Verbal Expert
Joined: 13 Aug 2009
Status: GMAT/GRE/LSAT tutors
Posts: 6920
Own Kudos [?]: 63659 [1]
Given Kudos: 1773
Location: United States (CO)
GMAT 1: 780 Q51 V46
GMAT 2: 800 Q51 V51
GRE 1: Q170 V170

GRE 2: Q170 V170
Send PM
Re: Since smoking-related illnesses are a serious health problem [#permalink]
1
Kudos
Expert Reply
jack0997 wrote:
Since smoking-related illnesses are a serious health problem in Normark, and since addiction to nicotine prevents many people from quitting smoking, the government of Normark plans to reduce the maximum allowable quantity of nicotine per cigarette by half over the next five years. However, reducing the quantity of nicotine per cigarette will probably cause people addicted to nicotine to smoke more cigarettes. Therefore implementing this plan is unlikely to reduce the incidence of smoking related illnesses.

Which of the following, if true, most strongly supports the argument about the consequences of implementing the Normarkian government's plan.

(A) Over half of the nonsmoking adults in Normark have smoked cigarettes in the past.

(B) most cigarettes currently sold in Normark contain somewhat less than the maximum amount of nicotine permitted by law.

(C) Inexpensive, smoke-free sources of nicotine, such as nicotine gum and nicotine skin patches, have recently become available in Normark.

(D) Many smokers in Normark already spend a large proportion of their disposable income on cigarettes.

(E) The main cause of smoking-related illnesses is not nicotine but tar in cigarette smoke.

VeritasKarishma GMATNinja

Are "nicotine skin patches" not a mode of smoking, thereby a strong case to consider option C?

Smoking is, by definition, "the action or habit of inhaling and exhaling the smoke of tobacco by sucking on the end of a lit cigarette, cigar, pipe, etc."

When you wear a nicotine skin patch, you do not inhale anything -- instead, the nicotine is absorbed through your skin. So, nicotine skin patches are not a method of smoking, but rather an entirely different method of getting your nicotine fix.

With this in mind, (C) doesn't support the argument that reducing the nicotine in cigarettes "is unlikely to reduce the incidence of smoking related illnesses." We have no idea how cheap nicotine patches will impact the sales of cigarettes, and we certainly can't conclude that cheap nicotine patches will not reduce smoking related illnesses.

I hope that helps!
Manager
Manager
Joined: 26 Apr 2016
Posts: 209
Own Kudos [?]: 48 [0]
Given Kudos: 6
GMAT 1: 640 Q44 V33
Send PM
Re: Since smoking-related illnesses are a serious health problem [#permalink]
I am not convinced with the right option E :

The question is asking for the support of the argument and not of the consequence- "Which of the following, if true, most strongly supports the argument about the consequences "

Argument : Reducing the quantity of nicotine per cigarette will probably cause people addicted to nicotine to smoke more cigarettes.

Consequence: Therefore implementing this plan is unlikely to reduce the incidence of smoking related illnesses

Now the probable correct options :

(B) most cigarettes currently sold in Normark contain somewhat less than the maximum amount of nicotine permitted by law.

(E) The main cause of smoking-related illnesses is not nicotine but tar in cigarette smoke.

I think that option B supports the Argument, whereas option C supports the the Consequence. In that case, the oprion B looks more correct option IMO.

Experts could you please chime in ?

MartyTargetTestPrep
GMATNinja
Target Test Prep Representative
Joined: 24 Nov 2014
Status:Chief Curriculum and Content Architect
Affiliations: Target Test Prep
Posts: 3480
Own Kudos [?]: 5137 [3]
Given Kudos: 1431
GMAT 1: 800 Q51 V51
Send PM
Re: Since smoking-related illnesses are a serious health problem [#permalink]
2
Kudos
1
Bookmarks
Expert Reply
abhishekmayank wrote:
I am not convinced with the right option E :

The question is asking for the support of the argument and not of the consequence- "Which of the following, if true, most strongly supports the argument about the consequences "

Argument : Reducing the quantity of nicotine per cigarette will probably cause people addicted to nicotine to smoke more cigarettes.

Consequence: Therefore implementing this plan is unlikely to reduce the incidence of smoking related illnesses

Now the probable correct options :

(B) most cigarettes currently sold in Normark contain somewhat less than the maximum amount of nicotine permitted by law.

(E) The main cause of smoking-related illnesses is not nicotine but tar in cigarette smoke.

I think that option B supports the Argument, whereas option C supports the the Consequence. In that case, the oprion B looks more correct option IMO.

Experts could you please chime in ?

While we generally call a question such as this one a Strengthen the ARGUMENT question, the correct answer will strengthen one particular thing, the support for the final conclusion of the argument.

So, in this case, we have to find a choice that supports the case for the prediction that you have called "the consequence."

Now, I understand that the question stem is not worded to clearly convey that that is what we have to do. It's worded in a way that could be taken as indicating that we have to somehow strengthen the internals of the argument. So, to confirm that (B) is not the correct answer, let's consider (B) just in case.

(B) indicates that cigarettes already contain less nicotine than is allowed by law. So, the implication of (B) is that reducing the maximum allowable quantity of nicotine may not have as much affect as it otherwise would have because the quantities of nicotine in cigarettes are already lower than the legal maximum.

However, (B) does not say that cigarettes currently contain half the quantity of nicotine currently allowed. In fact, (B) uses the word "somewhat." The word "somewhat" tends to convey that the difference between the allowable quantity of nicotine and the quantity of nicotine currently in cigarettes is not great, and that that difference is not as great as 50 percent, which would be a large difference.

Thus, (B) tends to indicate that, while the quantity of nicotine currently in cigarettes is "somewhat" smaller than the allowed maximum, it is greater than 50 percent of the allowed maximum. So, (B) tends to indicate that a 50 percent reduction could have an impact and, thus, that the plan could work. Thus, (B) does not support the author's conclusion that implementing the plan is unlikely to reduce the incidence of smoking related illnesses.

On the other hand, as you have agreed, (E) strengthens the support for the argument's main conclusion. So, (E) is the correct answer.

Originally posted by MartyTargetTestPrep on 02 Feb 2021, 12:30.
Last edited by MartyTargetTestPrep on 02 Feb 2021, 16:59, edited 1 time in total.
VP
VP
Joined: 14 Aug 2019
Posts: 1378
Own Kudos [?]: 846 [0]
Given Kudos: 381
Location: Hong Kong
Concentration: Strategy, Marketing
GMAT 1: 650 Q49 V29
GPA: 3.81
Send PM
Re: Since smoking-related illnesses are a serious health problem [#permalink]
MartyTargetTestPrep wrote:
abhishekmayank wrote:
I am not convinced with the right option E :

The question is asking for the support of the argument and not of the consequence- "Which of the following, if true, most strongly supports the argument about the consequences "

Argument : Reducing the quantity of nicotine per cigarette will probably cause people addicted to nicotine to smoke more cigarettes.

Consequence: Therefore implementing this plan is unlikely to reduce the incidence of smoking related illnesses

Now the probable correct options :

(B) most cigarettes currently sold in Normark contain somewhat less than the maximum amount of nicotine permitted by law.

(E) The main cause of smoking-related illnesses is not nicotine but tar in cigarette smoke.

I think that option B supports the Argument, whereas option C supports the the Consequence. In that case, the oprion B looks more correct option IMO.

Experts could you please chime in ?

While we generally call a question such as this one a Strengthen the ARGUMENT question, the correct answer will strengthen one particular thing, the support for the final conclusion of the argument.

So, in this case, we have to find a choice that supports the case for the prediction that you have called "the consequence."

Now, I understand that the question stem is not worded to clearly convey that that is what we have to do. It's worded in a way that could be taken as indicating that we have to somehow strengthen the internals of the argument. So, to confirm that (B) is not the correct answer, let's consider (B) just in case.

(B) indicates that cigarettes already contain less nicotine than is allowed by law. Notice that the implications of (B) are that reducing the maximum allowable quantity of nicotine may not have as much affect as it otherwise would have because the quantities of nicotine in cigarrettes are ALREADY lower than the legal maximum.

So, if anything (B) slightly weakens the case for concluding that reducing the legal maximum will have the predicted effect. In fact, for all we know given what (B) says, the amount of nicotine in cigarettes is already close to or at half the legal maximum. Thus, for all we know given what (B) says, reducing the legal maximum will have little to no effect.

So, regardless of how the question is worded, (B) cannot be the correct answer since, if anything, (B) weakens rather than strengthens the argument.

On the other hand, as you have agreed, (E) strengthens the support for the argument's main conclusion. So, (E) is the correct answer.


hello Sir MartyTargetTestPrep ,

I am not very clear how B weakens the argument instead of irrelevant.

say B has 85% nicotine and now nicotine can be reduced to 50%
X: 100% nicotine and reduce to 50% limit. means smokers may buy double the number of cigarettes
Y( as per our option B) has 85% nicotine and smokers may buy less than double the number of cigarettes( assume they spend money on buying same quantity of nicotine)

According to my thought, B become irrelevant because we don't know how many quantity they increase to .
If they keep same quantity of cigarettes , then plan will work. Means weaken the conclusion.
if smokers increase quantity then plan will not work. Plan will not work means strengthen the conclusion.
So without further information , B become irrelevant.

When i saw D option, it slightly weakness the conclusion because smokers are already spending a lot of money. assuming prices remain same, then they may not buy more cigarettes . It means plan would work . Plan work means weakness the conclusion.

Please clarify what I am missing for B then i am not able to see it is weakening the argument
Target Test Prep Representative
Joined: 24 Nov 2014
Status:Chief Curriculum and Content Architect
Affiliations: Target Test Prep
Posts: 3480
Own Kudos [?]: 5137 [1]
Given Kudos: 1431
GMAT 1: 800 Q51 V51
Send PM
Re: Since smoking-related illnesses are a serious health problem [#permalink]
1
Kudos
Expert Reply
imSKR wrote:
MartyTargetTestPrep wrote:
abhishekmayank wrote:
I am not convinced with the right option E :

The question is asking for the support of the argument and not of the consequence- "Which of the following, if true, most strongly supports the argument about the consequences "

Argument : Reducing the quantity of nicotine per cigarette will probably cause people addicted to nicotine to smoke more cigarettes.

Consequence: Therefore implementing this plan is unlikely to reduce the incidence of smoking related illnesses

Now the probable correct options :

(B) most cigarettes currently sold in Normark contain somewhat less than the maximum amount of nicotine permitted by law.

(E) The main cause of smoking-related illnesses is not nicotine but tar in cigarette smoke.

I think that option B supports the Argument, whereas option C supports the the Consequence. In that case, the oprion B looks more correct option IMO.

Experts could you please chime in ?

While we generally call a question such as this one a Strengthen the ARGUMENT question, the correct answer will strengthen one particular thing, the support for the final conclusion of the argument.

So, in this case, we have to find a choice that supports the case for the prediction that you have called "the consequence."

Now, I understand that the question stem is not worded to clearly convey that that is what we have to do. It's worded in a way that could be taken as indicating that we have to somehow strengthen the internals of the argument. So, to confirm that (B) is not the correct answer, let's consider (B) just in case.

(B) indicates that cigarettes already contain less nicotine than is allowed by law. Notice that the implications of (B) are that reducing the maximum allowable quantity of nicotine may not have as much affect as it otherwise would have because the quantities of nicotine in cigarrettes are ALREADY lower than the legal maximum.

So, if anything (B) slightly weakens the case for concluding that reducing the legal maximum will have the predicted effect. In fact, for all we know given what (B) says, the amount of nicotine in cigarettes is already close to or at half the legal maximum. Thus, for all we know given what (B) says, reducing the legal maximum will have little to no effect.

So, regardless of how the question is worded, (B) cannot be the correct answer since, if anything, (B) weakens rather than strengthens the argument.

On the other hand, as you have agreed, (E) strengthens the support for the argument's main conclusion. So, (E) is the correct answer.


hello Sir MartyTargetTestPrep ,

I am not very clear how B weakens the argument instead of irrelevant.

say B has 85% nicotine and now nicotine can be reduced to 50%
X: 100% nicotine and reduce to 50% limit. means smokers may buy double the number of cigarettes
Y( as per our option B) has 85% nicotine and smokers may buy less than double the number of cigarettes( assume they spend money on buying same quantity of nicotine)

According to my thought, B become irrelevant because we don't know how many quantity they increase to .
If they keep same quantity of cigarettes , then plan will work. Means weaken the conclusion.
if smokers increase quantity then plan will not work. Plan will not work means strengthen the conclusion.
So without further information , B become irrelevant.

When i saw D option, it slightly weakness the conclusion because smokers are already spending a lot of money. assuming prices remain same, then they may not buy more cigarettes . It means plan would work . Plan work means weakness the conclusion.

Please clarify what I am missing for B then i am not able to see it is weakening the argument

Now that you bring this up, I stand corrected. What (B) says suggests that the plan to reduce the amount of nicotine in cigarettes will in fact impact the amount of nicotine in cigarettes, and, thus, provides no additional support for the conclusion, while not weakening the case for the conclusion either.

Good catch.

Will update my response above to reflect.
GMAT Club Verbal Expert
Joined: 13 Aug 2009
Status: GMAT/GRE/LSAT tutors
Posts: 6920
Own Kudos [?]: 63659 [1]
Given Kudos: 1773
Location: United States (CO)
GMAT 1: 780 Q51 V46
GMAT 2: 800 Q51 V51
GRE 1: Q170 V170

GRE 2: Q170 V170
Send PM
Re: Since smoking-related illnesses are a serious health problem [#permalink]
1
Kudos
Expert Reply
abhishekmayank wrote:
I am not convinced with the right option E :

The question is asking for the support of the argument and not of the consequence- "Which of the following, if true, most strongly supports the argument about the consequences "

Argument : Reducing the quantity of nicotine per cigarette will probably cause people addicted to nicotine to smoke more cigarettes.

Consequence: Therefore implementing this plan is unlikely to reduce the incidence of smoking related illnesses

Now the probable correct options :

(B) most cigarettes currently sold in Normark contain somewhat less than the maximum amount of nicotine permitted by law.

(E) The main cause of smoking-related illnesses is not nicotine but tar in cigarette smoke.

I think that option B supports the Argument, whereas option C supports the the Consequence. In that case, the oprion B looks more correct option IMO.

Experts could you please chime in ?

MartyTargetTestPrep
GMATNinja

The passage is not divided up into an argument and consequences. Rather, the argument in the passage is about the consequences of implementing the Normarkian government’s plan.

In other words, the argument is not that reducing the nicotine per cigarette will cause people to smoke more cigarettes. Instead, the author is arguing that because the plan will cause people to smoke more cigarettes, it is unlikely to reduce the incidence of smoking related illnesses. So, the author is arguing about the consequences of the plan (i.e. saying the plan will not have its intended consequences).

The question asks us to find an answer choice that strengthens the author’s argument about the consequences. The author’s argument about the consequences is that the plan will likely not have its intended consequences. So, that is the argument that we are asked to weaken.

I hope that helps!
Intern
Intern
Joined: 07 Jul 2021
Posts: 39
Own Kudos [?]: 22 [0]
Given Kudos: 21
Concentration: Marketing, Strategy
GMAT 1: 640 Q44 V33
GMAT 2: 690 Q46 V38
GMAT 3: 720 Q49 V39
WE:Marketing (Manufacturing)
Send PM
Re: Since smoking-related illnesses are a serious health problem [#permalink]
WaitingSurprises wrote:
Since smoking-related illnesses are a serious health problem in Normark, and since addiction to nicotine prevents many people from quitting smoking, the government of Normark plans to reduce the maximum allowable quantity of nicotine per cigarette by half over the next five years. However, reducing the quantity of nicotine per cigarette will probably cause people addicted to nicotine to smoke more cigarettes. Therefore implementing this plan is unlikely to reduce the incidence of smoking related illnesses.

Which of the following, if true, most strongly supports the argument about the consequences of implementing the Normarkian government's plan.

(A) Over half of the nonsmoking adults in Normark have smoked cigarettes in the past.

(B) most cigarettes currently sold in Normark contain somewhat less than the maximum amount of nicotine permitted by law.

(C) Inexpensive, smoke-free sources of nicotine, such as nicotine gum and nicotine skin patches, have recently become available in Normark.

(D) Many smokers in Normark already spend a large proportion of their disposable income on cigarettes.

(E) The main cause of smoking-related illnesses is not nicotine but tar in cigarette smoke.


OA to follow after some discussion.



(A) Over half of the nonsmoking adults in Normark have smoked cigarettes in the past. Ok, so what? This doesnt do anything to the argument. Just information not required

(B) most cigarettes currently sold in Normark contain somewhat less than the maximum amount of nicotine permitted by law. If current ciggs have less nicotine then there is no need for the plan. But focus on Most and somewhat, a flimsy argument at best.

(C) Inexpensive, smoke-free sources of nicotine, such as nicotine gum and nicotine skin patches, have recently become available in Normark. That's good but it does not affect the success of the plan in any way. Out of scope, only talk about ciggs!

(D) Many smokers in Normark already spend a large proportion of their disposable income on cigarettes. Ok, so what? this has no effect on the plans success or failure

(E) The main cause of smoking-related illnesses is not nicotine but tar in cigarette smoke.Oh, now if the actual illness is caused by tar and not nicotine then wont smoking more ciggs with less nicotine exacerbate the issue (Tar will increase)? Nicotine not responsible>Tar is responsible> conclusion mentions smoking more ciggs so more tar. Directly validates the argument of the plan failing!
Senior Manager
Senior Manager
Joined: 08 Jan 2018
Posts: 297
Own Kudos [?]: 257 [0]
Given Kudos: 249
Location: India
Concentration: Operations, General Management
GMAT 1: 640 Q48 V27
GMAT 2: 730 Q51 V38
GPA: 3.9
WE:Project Management (Manufacturing)
Send PM
Re: Since smoking-related illnesses are a serious health problem [#permalink]
Outcome is-
The plan is not going to reduce the incidence of smoking related illness - as people are going to smoke more cigarettes

Strengthen

1. Wrong - about non smokers - telling a fact - but nothing about current situation

2. Wrong- Irrespective of the fact that they contain less than max, the amount is going to decrease after the plan, but how is it going to impact incidence of smoking related illness?? We don’t know

3. Wrong- so alternate inexpensive items are avl. Is money a concern for smokers ?? We don’t know. Irrespective of the fact that inexpensive item are avl., people may or may not continue to buy.

4. Wrong- we don’t know whether they are going to stop or continue spending- and therefore buying cigarettes

5. Right - Now if tar is the cause of disease , and we are reducing nicotine , then even after reducing the content and without increasing the consumption, the incidence of disease is not going to go down, as we are not targeting the root cause of the disease

Posted from my mobile device
Senior Manager
Senior Manager
Joined: 16 Oct 2020
Posts: 265
Own Kudos [?]: 163 [0]
Given Kudos: 2385
GMAT 1: 460 Q28 V26
GMAT 2: 550 Q39 V27
GMAT 3: 610 Q39 V35
GMAT 4: 650 Q42 V38
GMAT 5: 720 Q48 V41
Send PM
Re: Since smoking-related illnesses are a serious health problem [#permalink]
If anything, option C is a weakener as it alludes to a potential reduction in smoking. Because inexpensive, smoke-free sources of nicotine have become available, smokers can (more easily than before) get their nicotine fix even without smoking.

In other words, reducing the quantity of nicotine per cigarette will probably not cause people addicted to nicotine to smoke more cigarettes (as they can get their fix in other ways). Therefore implementing this plan is likely to reduce the incidence of smoking related illnesses.
Intern
Intern
Joined: 02 Jul 2023
Posts: 10
Own Kudos [?]: 4 [0]
Given Kudos: 21
Send PM
Re: Since smoking-related illnesses are a serious health problem [#permalink]
MartyTargetTestPrep. please help

Nowhere has it been mentioned that consuming nicotine causes smoking - related illness. Presence of nicotine in cigs only keeps the person addicted to smoking. Then why is option E correct?
Director
Director
Joined: 17 Aug 2009
Posts: 624
Own Kudos [?]: 31 [0]
Given Kudos: 21
Send PM
Re: Since smoking-related illnesses are a serious health problem [#permalink]
Understanding the argument -
Since smoking-related illnesses are a serious health problem in Normark, and since addiction to nicotine prevents many people from quitting smoking, the government of Normark plans to reduce the maximum allowable quantity of nicotine per cigarette by half over the next five years. - There is a plan and two causes for the plan. The goal is to reduce smoking-related illnesses.
However, reducing the quantity of nicotine per cigarette will probably cause people addicted to nicotine to smoke more cigarettes. - contrast with "however," and the plan seems to have the opposite of the intended effect.
Therefore implementing this plan is unlikely to reduce the incidence of smoking related illnesses. - Conclusion

We have to strengthen the conclusion. What conclusion? "This plan is unlikely to reduce the incidence of smoking-related illnesses."

Option Elimination -

(A) Over half of the nonsmoking adults in Normark have smoked cigarettes in the past. - how does it even relate to the scope of the argument, which is to strengthen "this plan is unlikely to reduce the incidence of smoking-related illnesses." "non-smoking adults"? We are worried about smokers who still smoke. Out of scope.

(B) most cigarettes currently sold in Normark contain somewhat less than the maximum amount of nicotine permitted by law. - The argument talks about reducing by 50%, and this option talks about "somewhat." It can be 1% less than the maximum or 20% less, but a 50% reduction is still an advantage. Good distortion.

(C) Inexpensive, smoke-free sources of nicotine, such as nicotine gum and nicotine skin patches, have recently become available in Normark. - This is out of scope. As our scope is limited to the effect of Nicotine and health issues related to smoking. How is it even related to the scope of the argument, which is to strengthen "this plan is unlikely to reduce the incidence of smoking-related illnesses." If we become too obsessed with this option, make a lot of assumptions, and somehow reach a conclusion that it can help reduce smoking side effects, then, at best, this is the opposite of what we are looking for.

(D) Many smokers in Normark already spend a large proportion of their disposable income on cigarettes. - At best, if they can't buy more, then nicotine side effects may be reduced, and it'll weaken the conclusion. We are looking for a strengthener here and not a weakener.

(E) The main cause of smoking-related illnesses is not nicotine but tar in cigarette smoke. - So, as people consume more boxes, they inhale more tar, they harm their health, and the plan fails, which strengthens the conclusion.
GMAT Club Bot
Re: Since smoking-related illnesses are a serious health problem [#permalink]
Moderators:
GMAT Club Verbal Expert
6920 posts
GMAT Club Verbal Expert
238 posts
CR Forum Moderator
832 posts

Powered by phpBB © phpBB Group | Emoji artwork provided by EmojiOne